Difference between revisions of "2011 AMC 12B Problems/Problem 17"

(Solution)
 
(6 intermediate revisions by 2 users not shown)
Line 16: Line 16:
 
Assume <math>h_{n}(x)=10^n x - (1 + 10 + 10^2 + ... + 10^{n-1})</math> is true for n:
 
Assume <math>h_{n}(x)=10^n x - (1 + 10 + 10^2 + ... + 10^{n-1})</math> is true for n:
  
\[\begin{align*}
+
<cmath>\begin{align*}
 
h_{n+1}(x)&= h_{1}(h_{n}(x))\\
 
h_{n+1}(x)&= h_{1}(h_{n}(x))\\
 
&=10 h_{n}(x) - 1\\
 
&=10 h_{n}(x) - 1\\
Line 22: Line 22:
 
&= 10^{n+1} x - (10 + 10^2 + ... + 10^{n}) - 1\\
 
&= 10^{n+1} x - (10 + 10^2 + ... + 10^{n}) - 1\\
 
&= 10^{n+1} x - (1 + 10 + 10^2 + ... + 10^{(n+1)-1})
 
&= 10^{n+1} x - (1 + 10 + 10^2 + ... + 10^{(n+1)-1})
\end{align*}\]
+
\end{align*}</cmath>
  
 
Therefore, if it is true for n, then it is true for n+1; since it is also true for n = 1, it is true for all positive integers n.
 
Therefore, if it is true for n, then it is true for n+1; since it is also true for n = 1, it is true for all positive integers n.
Line 29: Line 29:
  
 
The sum of the digits is 8 times 2010 plus 9, or <math>\boxed{16089\textbf{(B)}}</math>
 
The sum of the digits is 8 times 2010 plus 9, or <math>\boxed{16089\textbf{(B)}}</math>
 +
 +
==Solution 2 (Quick, Non-Rigorous Trends)==
 +
As before, <math>h_1(x)=10x-1</math>. Compute <math>h_1(x)</math>, <math>h_2(x)</math>, and <math>h_3(x)</math> to yield 9, 89, and 889. Notice how this trend will repeat this trend (multiply by 10, subtract 1, repeat). As such, <math>h_{2011}</math> is just 2010 8's followed by a nine. <math>2010(8)+9=\boxed{\textbf{B)}16089}</math>.
 +
 +
~~BJHHar
  
 
== See also ==
 
== See also ==
 
{{AMC12 box|year=2011|num-b=16|num-a=18|ab=B}}
 
{{AMC12 box|year=2011|num-b=16|num-a=18|ab=B}}
 
{{MAA Notice}}
 
{{MAA Notice}}

Latest revision as of 19:36, 7 August 2023

Problem

Let $f(x) = 10^{10x}, g(x) = \log_{10}\left(\frac{x}{10}\right), h_1(x) = g(f(x))$, and $h_n(x) = h_1(h_{n-1}(x))$ for integers $n \geq 2$. What is the sum of the digits of $h_{2011}(1)$?

$\textbf{(A)}\ 16081 \qquad \textbf{(B)}\ 16089 \qquad \textbf{(C)}\ 18089 \qquad \textbf{(D)}\ 18098 \qquad \textbf{(E)}\ 18099$

Solution

$g(x)=\log_{10}\left(\frac{x}{10}\right)=\log_{10}\left({x}\right) - 1$

$h_{1}(x)=g(f(x))\text{ = }g(10^{10x}=\log_{10}\left({10^{10x}}\right){ - 1 = 10x - 1}$

Proof by induction that $h_{n}(x)\text{ = }10^n x - (1 + 10 + 10^2 + ... + 10^{n-1})$:

For $n=1$, $h_{1}(x)=10x - 1$

Assume $h_{n}(x)=10^n x - (1 + 10 + 10^2 + ... + 10^{n-1})$ is true for n:

\begin{align*} h_{n+1}(x)&= h_{1}(h_{n}(x))\\ &=10 h_{n}(x) - 1\\ &=10 (10^n x   - (1 + 10 + 10^2 + ... + 10^{n-1})) - 1\\ &= 10^{n+1} x - (10 + 10^2 + ... + 10^{n}) - 1\\ &= 10^{n+1} x - (1 + 10 + 10^2 + ... + 10^{(n+1)-1}) \end{align*}

Therefore, if it is true for n, then it is true for n+1; since it is also true for n = 1, it is true for all positive integers n.

$h_{2011}(1) = 10^{2011}\times 1{ - }(1 + 10 + 10^2 + ... + 10^{2010})$, which is the 2011-digit number 8888...8889

The sum of the digits is 8 times 2010 plus 9, or $\boxed{16089\textbf{(B)}}$

Solution 2 (Quick, Non-Rigorous Trends)

As before, $h_1(x)=10x-1$. Compute $h_1(x)$, $h_2(x)$, and $h_3(x)$ to yield 9, 89, and 889. Notice how this trend will repeat this trend (multiply by 10, subtract 1, repeat). As such, $h_{2011}$ is just 2010 8's followed by a nine. $2010(8)+9=\boxed{\textbf{B)}16089}$.

~~BJHHar

See also

2011 AMC 12B (ProblemsAnswer KeyResources)
Preceded by
Problem 16
Followed by
Problem 18
1 2 3 4 5 6 7 8 9 10 11 12 13 14 15 16 17 18 19 20 21 22 23 24 25
All AMC 12 Problems and Solutions

The problems on this page are copyrighted by the Mathematical Association of America's American Mathematics Competitions. AMC logo.png